Instigator / Pro
15
1485
rating
6
debates
50.0%
won
Topic
#55

US voters should have to pass a citizenship test in order to be able to vote, but only for Senate and Presidential candidates

Status
Finished

The debate is finished. The distribution of the voting points and the winner are presented below.

Winner & statistics
Better arguments
3
6
Better sources
6
6
Better legibility
3
3
Better conduct
3
3

After 3 votes and with 3 points ahead, the winner is...

RationalMadman
Parameters
Publication date
Last updated date
Type
Standard
Number of rounds
4
Time for argument
Three days
Max argument characters
30,000
Voting period
Two months
Point system
Multiple criterions
Voting system
Open
Contender / Con
18
1687
rating
555
debates
68.11%
won
Description

I believe and shall be arguing in favor of the idea that US voters should have to pass a citizenship test in order to be able to vote, but only for Senate and Presidential candidates. This means that other elections on the county and district levels would not require voters to have passed a citizenship in order for them to cast ballots, for reasons detailed in the arguments section. First round acceptance only, Conclusions in the final rounds

Round 1
Pro
#1
Evidently the short description made when making the debate does not become the opening round starting argument. 

First round acceptance only, arguments begin in round 2, continue in round 3, and debate gets concluded in round 4 
Con
#2
I accept, or do I need to pass a debate site membership test to have the right to do so? ;)
Round 2
Pro
#3
Alright so for this debate my arguments will be split into 2 main branches

Branch 1) Why having voters be able to pass a citizenship test to be able to vote is more preferable to the current status quo
Branch 2) Why this additional qualification should only be applied to voters for Senate and Presidential elections

Lets begin

========================================================================

Arguments for why voters should have to pass a citizenship test to be able to vote

1A) The average voter in US elections can be alarmingly stupid, and requiring voters to pass a citizenship test would weed out the alarmingly stupid ones

If you listen to the news you'll hear every once in a while a headline that says "___% of Americans dont know about ____". The most recent examples I could find include:

2/3rds of Millennials dont know what Auschwitz is: https://www.cbsnews.com/news/holocaust-study-millennials/
1/4th of Americans think the Sun revolves around the Earthhttp://time.com/7809/1-in-4-americans-thinks-sun-orbits-earth/
1/3rd of Americans cannot name ONE branch of the government: https://www.cnn.com/2017/09/13/politics/poll-constitution/index.html
1/3rd of Americans cannot think of a freedom protected by the First Amendment (Same link as previous)

These Americans, who know nothing about what should be basic fundamental knowledge about America and the world, have just as much power with their vote as someone who DOES know what Auschwitz is, knows that the Earth revolves around the Sun, knows what the first amendment is, and knows at least one of the branches of the US government. A smarter and more knowledgable voter base would naturally result in the election of smarter and more knowledgable politicians and leaders, who could stay calmer in tense situations, be craftier in negotiating trade deals or embargoes with other nations, plunge into or avoid military conflicts, and be more gifted at enacting legislation that would benefit the country both short term and long term.... Having a voter base that is alarmingly ignorant of basic facts of the world and their own government on the other hand limits the quality of politicians who get elected, and could possibly jeopardize the safety and health of the entire world if they are conned into supporting a bad politician. 

There are ignorant voters who do not know basic facts about the world who are casting votes that could alter its fate. If these voters are weeded out by failing to pass a citizenship test, then the influence that these people would have in electing politicians would be substantially reduced, and everyone would be better off for it. 



1B) The US is too powerful in the world to afford being led by bad politicians for an extensive period of time

The US President and US senators have incredible power in altering the state of the world and change the course of the nations history, so its reasonable to make sure that voters who elect these people also know basic facts about the state of the world and US history before allowing them to vote. As an example, a country like Costa Rica or Haiti could afford to have bad leaders elected by an ignorant voter base because those countries have an incredibly small impact on world events and world history, due to their small size, small wealth, and weak military power.... The United States on the other hand is one of the largest countries in the world, has the biggest economy in the world, and has the strongest military in the world (https://www.washingtonpost.com/news/wonk/wp/2014/07/03/21-maps-and-charts-that-prove-america-is-number-one/?utm_term=.bd30e44bcac0). Bad leaders from the US are far more likely to cause harm in the rest of the world than bad leaders of smaller countries like Costa Rica, because of how much of a larger role the US plays in world affairs.

If we operate on the assumption that a smarter voter base would lead to better politicians being elected to office, then the risk of the US plunging the world into an avoidable crisis would drop, which is in the best interest of everyone given how capable the US is at impacting world events



1C) A citizenship test is a fair, unbiased assessment of world knowledge and American affairs

In order to avoid potential bias from a test with questions that could target one side of the political spectrum, while at the same time actually testing a persons knowledge about American affairs to a good degree, the citizenship test given to immigrants wishing to become citizens is a natural solution. Current citizenship tests (link here: http://www.citizenshipstudyguide.com/member/citizenship-test-questions-quiz.php ) include unbiased but good to know questions that immigrants have to know in order to become citizens. Questions include who is the current VP, Why does the flag have 50 stars, Presidential terms last how many years, etc. that are all fairly basic questions that do not single out one particular party or ideology. Creating an entirely different test from scratch risks questions becoming too specific for the average voter to know about, or to biased against a particular set of beliefs.


========================================================================

Arguments for why this policy should only be applied for Senate and Presidential elections

2A) Presidents and Senators have the biggest ability in the US to impact policy and world events. 

The US President has tremendous power to pass or halt domestic legislation and also decide to intervene in foreign affairs or not. One decision, one vote. Thats all it takes..... The US president is arguably the most powerful elected position in the history of the entire world, so if ANY electoral candidate should be elected based on voters who could actually pass a citizenship test, it would be the US president.  

After the President, Senators have the biggest individual impact on US domestic and foreign affairs. Since 1980, a total of 28 Bills, Acts, and Motions have been decided by a single vote in the Senate, an average of almost 1 per year ( https://www.senate.gov/pagelayout/reference/four_column_table/Tie_Votes.htm ) According to the same site, since 1789, there have been 263 cases where legislation was decided by a single vote, also averaging out to about 1 bill per year. Imagine how differently those acts would have turned out if only informed citizens elected senators. President Andrew Johnson avoided being impeached by a single vote back in 1868, imagine how things could have turned out differently if senators were elected only by well informed voters. ( https://www.senate.gov/artandhistory/history/minute/The_Senate_Votes_on_a_Presidential_Impeachment.htm )

The President of course has incredible power in deciding the fate of important legislation that could alter the fate of the country and even the world. The Senate comes in second place, with individual governors of particularly large states coming in third. If the president and US senators are the people in power most capable of altering national or world history, they should be the ones if anyone that should have their supporters be required to pass citizenship tests. 



2B) The (small) Senate plays a role in the appointment of judges and advisers to high level government positions, while the (large) House of Representatives does not. 

In 2018 alone, there were NINE votes decided by one Senator regarding the appointment of certain officials to federal positions ( https://www.govtrack.us/congress/votes#chamber[]=1&sort=margin ) In 2017 there were EIGHTEEN different votes decided by just 1 Senator, but those admittedly ranged from appointing advisers to rule changes of existing legislation to the actual passage of new legislation. 

In comparison, the House of Representatives only had ONE vote decided by a single vote in 2018 (Link located in argument 2C). Based on the previous argument and this new argument;

- Because the Senate plays a role in the appointment of people to federal positions while the House does not,
- Because there have been more 1 vote decisions made in the Senate than in the House,
- Because there are fewer Senators (100) than there are Representatives (435)

An individual Senator has far more power and ability to impact US history compared to an individual Representative of the House. For that reason, it is reasonable to require voters to pass citizenship tests for electing senators while exempting that requirement for electing Representatives. 



2C) Members of the House of Representatives are rarely the deciding factor in determining whether legislation passes or not

The House of Representatives, dating back to 2010, has passed or rejected bills by a single vote 15 times (1 in 2018, 3 times in 2016, 1 in 2015, 1 in 2014, 2 in 2013, 2 in 2012, 3 in 2011, and 2 in 2010 https://www.govtrack.us/congress/votes#session=294&chamber[]=2&sort=margin ) That comes out to an average of two 1-vote decisions per year. Of those 15 instances though, 1 was to call an early recess for the year (2010), 6 were votes on adding amendments to bills rather than votes on entire bills themselves (all three in 2011, both in 2012, one in 2014) and most of the remaining 8 were votes to amend already enacted legislation (all votes from 2015-2018) 

The House of Representatives has a reduced role in deciding legislation that impacts the country, and in the few instances that votes do come down to a single vote, it is usually just for a vote to include an amendment to a bigger piece of legislation, or to amend laws that have already been enacted. 



2D) State governors only ever impact domestic issues rather than have a hand in deciding international affairs, and only governors from large states impact the lives of many people. 

State governors, like House Representatives and Senators, are elected only by people of an individual state. Unlike both House Representatives and Senators though, the powers that a state governor has is limited entirely to domestic affairs, having almost no impact on international issues. On top of that, state governors's influence is largely limited to their own individual state, and maybe states that border them, whereas House representatives can enact legislation that impacts the country as a whole. 

On top of that, a governor of a small, unpopulated state has even an even smaller impact on national or international events simply because the number of people their actions impact is reduced. The governor of Texas has a far greater ability to impact people's lives than the governor of Montana, though both do not have as much of a say in national and international affairs as a Senator from either of those states as well. 

Because governors have zero say in foreign affairs, and almost zero say in domestic affairs outside of their own states, a state governor does not have the capability of altering US history or World history compared to the President and Senators. It therefore is reasonable that governors should also be exempted from being elected by only voters who have passed a citizenship test like members of the House of Representatives, due to the smaller impact they have on people in comparison. 



2E) City and county level political positions have almost no impact whatsoever

The final tier of elected officials are at the city and county level, whose impact almost rarely impacts anyone beyond the city limits or country lines. There isn't much of a reason to hold the mayor of Ceder Key (population 700) to the same voter intelligence standards as the President of the United States. 

=================================================================================================

(Just as a footnote, in the event that a voter does not pass a citizenship test, there would be opportunities made available for them to retake the test. How soon/frequently/many time they would be allowed to do that could be left up to states to decide..... Those who do not pass a citizenship test would receive a different ballot for primaries and election days that do not include options to vote for presidents or senators..... I say this because I would strongly prefer this debate be centered around the ethics and reasons for/against voters being required to pass citizenship tests, rather than semantical arguments about how this idea would be handled bureaucratically)
Con
#4
Prop = Propostion = Imabench
Con = Contender = RationalMadman
S&G: I will be using British English outside of quotes.

To understand why Pro is wrong, we need to begin to understand what voting rights are and what the law of US has as the core way of determining who should vote and who should not. From those 'core values' that we cannot question, we then can question whether or not Pro's suggested regime is closer to the core US law's values regarding voting rights or whether mine is.


A big error that a lot of debaters on my side would make (and you yourself as a reader may be thinking) is that the angle one should take regarding voting is to look at the Constitution and original basis of law itself. The issue with this is you're going to find that the Constitution is extremely crafty or at least was prior to added amendments. The origin of law in the US was one that was designed exclusively (intentionally) to mean that politicians who benefitted 'white' male land owners (the invading race and sub-sets, back then light skinned Spaniard race was considered white and not 'Latino' so it's not inherently Caucasian). This does support my side as it in no way at all has any basis in knowledge or IQ (I will get to the lack of separation between knowledge and intelligence later on with the case Pro has made). What matters more is the additional legislation regarding voting that has been enacted on top of the Constitution. This will be how I begin to explain that this resolution flies in the face of the US law's clear path towards (and even away from) a voting system of segregation and towards one of maximal unification so long as it is believed that the voters are capable of making decisions for themselves. Notice I said the key is the ability to make decisions and not the ability to make well-informed ones? I will prove first by raw values displayed in US legislation that this resolution defies US core and even non-core values in blatant ways. After that, I will go into the specific Citizenship Test and all the flaws with using that as the buffer.

So, let's go on a journey to what really matters in voting.

Full credit to https://www.infoplease.com/timelines/us-voting-rights for helping me with the timeline and mapping out what I'm presenting here.

Alright, in 1776 there was basically no real mention of the specifics of Voting or what makes it a fundamental Right in the Constitution. There was basically a system where 'white' male land owners could vote (and even vote proportional to the land they owned I am quite certain). The reason there is so little about this time is that it was basically intentionally not heavily made public or apparent how it worked and those it ignored and/or oppressed were kicked to the curb. From 1776 to 1870 the resentment in the non-white-male-land-owners of the US had grown and this happens at the culmination:


The 15th Amendment to the Constitution granted African American men the right to vote by declaring that the "right of citizens of the United States to vote shall not be denied or abridged by the United States or by any state on account of race, color, or previous condition of servitude." Although ratified on February 3, 1870, the promise of the 15th Amendment would not be fully realized for almost a century. Through the use of poll taxes, literacy tests and other means, Southern states were able to effectively disenfranchise African Americans. It would take the passage of the Voting Rights Act of 1965 before the majority of African Americans in the South were registered to vote.


So, far Pro could still make his case, sure. It's unclear if the journey to reduce restrictions as an ethos of US law regarding voting is one that will keep snowballing from this time on right? Well, let's see how the snowball rolls or undoes itself (clue: it ends up rolling hard after some setbacks).

Let's rewind a second and concede something to Pro. Back in 1855, the beginning of literacy tests as a means to restrict voters was invented. The U.S. Citizenship is not a literacy test (at all) and doesn't test linguistic capabilities beyond studying a set of predetermined questions but I'll go into that later.

As for what I just conceded it is defeated long before present day (August 26th-27th 2018) because:

In 1970, Congress voted to extend renewable portions of the Voting Rights Act for five more years. They also added some new provisions to the act.

The 1970 amendments included a nationwide ban on literacy tests and reduced residency requirements that could be applied in presidential elections.

The 1970 reauthorization also reduced the voting age [link to AGE subpage] in national elections from 21 to 18 years of age. Though this provision was overturned by the U.S. Supreme Court for non-federal elections, the voting age was permanently lowered by passage of the 26th Amendment in 1971.

^ If Pro has any question on the truth of what I stated here I will happily prove the 26th Amendment and reduced residency requirements really happened and did causes these 2 things in Round 2.

So, let's return to the snowball... After 1870 what happens next? Well, a lot of states begin to bring in literacy tests and poll taxes as it becomes much more like Pro's ideal society in terms of how it fights off people they view as unworthy to vote. That is, it keeps moving in that direction until the snowball returns with a vengeance. It begins growing bigger again by the following being passed in 1913:

17th Amendment

The Senate of the United States shall be composed of two Senators from each state, elected by the people thereof, for six years; and each Senator shall have one vote. The electors in each state shall have the qualifications requisite for electors of the most numerous branch of the state legislatures.

When vacancies happen in the representation of any state in the Senate, the executive authority of such state shall issue writs of election to fill such vacancies: Provided, that the legislature of any state may empower the executive thereof to make temporary appointments until the people fill the vacancies by election as the legislature may direct.

This amendment shall not be so construed as to affect the election or term of any Senator chosen before it becomes valid as part of the Constitution.
In fact, this direction of growth is later added to and as an indirect result of it in 1964:

Amendment 24 [source says XXIV but this is numerically same as 24]

ABOLITION OF POLL TAXES
Passed by Congress August 27, 1962. Ratified January 23, 1964

SECTION 1

The right of citizens of the United States to vote in any primary or other election for President or Vice President, for electors for President or Vice President, or for Senator or Representative in Congress, shall not be denied or abridged by the United States or any State by reason of failure to pay poll tax or other tax.

SECTION 2

The Congress shall have power to enforce this article by appropriate legislation.


Which has a lot to do with Reynolvds vs Sims (1964) which in turn leads to:

One-Person, One-vote Rule

The rule that, under the Equal Protection Clause of the Constitution, legislative voting districts must be the same in population size. The idea behind the rule is that one person’s voting power ought to be roughly equivalent to another person’s within the state. See Reynolds v. Sims, 377 U.S. 533 (1964).

Can we stop here? Basically we can, the other details are not so relevant to the resolution.

I'd like to now just point out that while the age of drinking and gambling are age 21, the age of voting is 18. This strongly implies that the US views voting as something that isn't the same an informed decision requiring incredible wisdom and self control but simply one that once you can consent and are a citizen you can vote (yes Pro is going to make citizens have to do the Citizenship Test, that's the whole reason Con is the right side to be on).

I will now like to say something regarding the specifics of the US Citizenship Test and how irrelevant what Pro points is to it. 

Firstly, Pro is confusing knowledge for intelligence and I will prove this in Round 2 if he doesn't explain why he's done this, I will fully elaborate on why this alone is going to destroy the case he is making.

Secondly, Pro has mistaken knowledge as something the Citizenship Test is made to test in terms of voting knowledge. The correct name for it is a Naturalisation Test but the reason that Pro doesn't use this name is that it would highlight the contradiction of making US citizens have to take it since they are already citizens who don't need to be naturalised.

Let's just clarify two things. The purpose of this test and the questions in it are not going to ask things about Auschwitz to my knowledge or even test if the one taking it is a Flat-Earther (although it may mention NASA and ask what it is since it's a Federal thing) it is there to do the following:

The Naturalization Test

To become a naturalized U.S. citizen, you must pass the naturalization test. At your naturalization interview, you will be required to answer questions about your application and background.

Now, did you know that part of the Citizenship test is an English (which means Literacy if you interpret it properly) Test? That would mean it violates the 1970 amendments directly.

While it is required you know English in order to become a Citizen, that is something we can argue in another debate. What is clear is if you are already a Citizen not only do the 1970 Amendments help support Con but the 1975 even more explicitly make what Pro is suggesting fly straight in the face of US law ethos in how it is has evolved Voting Rights.
The 1975 Amendments extended the provisions of the Voting Rights Act of 1965 for seven years. Established coverage for other minority groups including Native Americans, Hispanic Americans, and Asian Americans. Permanently banned literacy tests. Passed by the 94th Congress (1975–1977) as H.R. 6219.

So an entire half of the Citizenship test is a literacy test but if you are a Citizen you should lose your voting rights if you don't pass a test that clearly over time has been established by US society and its lawmakers (society-to-lawmaker via democracy) as being immoral or at least completely illegal makes sense? I don't think so.

The English test has three components: reading, writing, and speaking. Your ability to speak English will be determined by a USCIS Officer during your eligibility interview on Form N-400, Application for Naturalization. For the reading portion, you must read one out of three sentences correctly. For the writing test, you must write one out of three sentences correctly.
^ Same source just something more relevant to the case of Con:
Certain applicants, because of age and time as a permanent resident, are exempt from the English requirements for naturalization and may take the civics test in the language of their choice.

On top of that, the other part of the Citizenship Test has absolutely nothing to do with intelligence, it tests for knowledge that sure, is partly to do with Politics but do not in any way imply the one who gets better percentage is a more intelligent voter.

Something I want to quickly establish here is that loyalty has nothing to do with intelligence, what Pro is actually advocating is to test for intelligence by testing for loyalty and then using a test that tests solely for knowledge.

Loyalty =/= Intelligence =/= Knowledge

I will leave this as it is for now and let Pro defend their case but I reiterate: Nearly every single thing that Pro said most voters don't know in Round 1 is not tested for in the US Citizenship/Naturalisation Test.
Round 3
Pro
#5
Con breaks down his response to the arguments I made in the previous round into 2 branches of his own. The first branch revolves around examining what voting is 'really about' in the US and why my resolution allegedly violates that, and the second branch of arguments focuses on whether or not a US citizenship test would be effective at screening out unintelligent voters. 

The first half of Con's arguments in round 2 revolve around theoretical opinions for what he believes 'really matters' in voting in the United States..... He lists several pieces of legislation in US history along the way, but not as evidence for why this idea proposed in the resolution would be illegal. Instead he uses the legislation cited in his argument simply to pinpoint times in US history when voting practices shifted towards an 'ultimate goal' of:

'Maximal unification so long as it is believed that the voters are capable of making decisions for themselves'.
So for those of you who may not have understood Con's overall defense against the resolution in round 2, Con has opted to argue against the resolution primarily on the belief of what voting should 'be about' in the US..... In doing this: 

- Con concedes all the arguments from branch 2 of my opening arguments about why the requirement of having voters be able to pass a citizenship test should be limited to voting just for Presidents and Senators.

That leaves us with the main arguments of the debate being a moral argument of what US voting standards should be about, as evidenced by Con's other statement:

"The US views voting as something that isn't the same an informed decision requiring incredible wisdom and self control but simply one that once you can consent and are a citizen you can vote"
In response to this being the new focus of the debate, I shall introduce additional arguments about 1) What US voting laws should seek to achieve, 2) Introduce evidence of the US restricting voting rights beyond people being able to simply be capable of making a decision, and 3) Argue that the curtailing of US voting rights in order for the government to be better able to carry out other US views.

I shall start with that last one first:

.
.
.
.
.

============================================================================

1A) Restricting voting rights of uninformed voters enables the US government to better pursue/defend its other core ideals and beliefs

When you think of 'American ideals' and other 'core' things that the US likes to claim it stands for/is the protector of, a few things can come to mind.... "Defending Democracies overseas", "Defending Human Rights", "Freedom from Tyranny", "Freedom of Speech and Religion", "Right to Defend oneself" are all ones I can think of off the top of my head. You can probably think of many more that I haven'e even listed. 
 
All of these things can be jeopardized when uninformed voters are able to sway the outcome of a close contest to elect worse politicians the nation would otherwise get..... A voter who thinks the sun orbits around the Earth could conceivably believe that global warming is a lie as well, and therefore vote for a politician who wants to drastically expand government surveillance laws, simply because that politician said in a commercial that he also doesn't believe in global warming..... A voter who doesn't know what the 1st Amendment is could vote for a politician who wants to ban all mosques just because the politician talks the same way he does...... A voter who only listens to Alex Jones for news may vote for a politician that would outlaw gay marriage just because that candidate is a man and the other he is facing is a woman. 

Voters can make decisions on who to vote for based on incredibly stupid, uninformed, under-informed, and even hateful reasons, whch can lead to the election of candidates that would undermine the core values the US stands for. Many (not all) exceptionally stupid voters can be weeded out from having a say in senate and presidential elections just by requiring them to pass a citizenship test in order to vote for them (presidents and senators). This resolution wouldn't eliminate ALL bad voters or screen out ALL bad decisions people use for electing someone, but it would undoubtably reduce the number of times those votes are made, which would increase the number of better politicians the US gets who would better stand up for and defend the United States' core ideals and beliefs. 

If the argument against restricting voting ability of bad voters is that its 'un-American', then it is overruled by the fact that better politicians elected by a smarter voter base would be better able to defend 'pro-American' beliefs. Whether or not the claim restricting voting ability of bad voters goes against what core US voting beliefs stand for brings us to our next argument

.
.
.
.
.

1B) Both state and the US governments have restricted voting rights for people beyond just being able to make a decision for themselves and being citizens. 

There are laws in the US that prevent mentally ill people from being able to vote, which based on Con's argument of what voting in the US is 'really about' is acceptable, since he believes the bar is set at 'being able to make up their mind and being a citizen'.... The following is a list of voting rights limitations the US government has also enforced that go BEYOND a person being able to make a decision for themselves or being citizens. 

- Voters must have identification cards in order to cast a vote 

- Voters must be registered voters in order to vote

- States have the right to bar people convicted of crimes from voting (even if its a misdemeanor) 

- (Supreme Court ruled this) States have the right to revoke voter registration for those who have not voted in previous elections https://www.usatoday.com/story/news/politics/2018/06/11/supreme-court-states-purge-voters-who-dont-vote/587316002/  

- States have the right to redistrict districts to undermine the say from voters of a political party

If Cons belief that the core of the US attitude towards voting revolved around a person just being able to consent and be a citizen, then these other parameters that state governments, the national government, and even the Supreme Court have upheld as constitutional would not be in place..... Instead, we have many restrictions beyond just being able to consent and be a citizen in order to vote. You have to have some form of ID in order to vote, you have to have registered in order to vote, you cant have committed a felony or in some cases any crime at all in order to vote, you could have your voting rights removed if you didn't vote last time, and your vote could be limited based on your political affiliation thanks to US states being allowed to gerrymander. 

If its not true that the US's core beliefs towards voting are aimed exclusively at people just being able to consent and be a citizen, then It is not inherently 'un-American' for US voting laws to require people to pass a citizenship test, since other laws that restrict voters beyond just being citizens who can give consent already exist, and for good reason.  

.
.
.
.
.

1C) What is the core US belief towards voting if it goes further then just being able to give consent and be a citizen?

If con's claim that US voting beliefs revolve around citizenship and ability to give consent doesn't hold water, then what is the US's overall belief regarding voting? Its arguably a variety of different things, which would vary from person to person based mostly on their own subjective opinions. 

The existence of identification laws and registration laws, which can be abused by some states, exists for the most part to keep voter fraud to non-existent levels, which I think most people can make peace with. Being able to give consent and being a citizen in order to vote is universally agreed upon. Then comes gerrymandering and voter registration purges, which a lot of people are against but the US Supreme Court has allowed/upheld in the past. 

What is legal in terms of voting rights and what the people of the US like/approve of are not always one in the same, but it is the sometimes unpopular rulings of the Supreme Court on voters rights which could give us the clearer picture of what US core values of voting are really like according to the Constitution of the United States, which brings me to the second half of Con's arguments.

.
.
.
.
.

============================================================================

The second branch of Con's arguments against the resolution revolve around the claim that having voters pass citizenship tests does not equate to them being wise or knowledgable, and mainly that a citizenship test is the same as a literacy test which are expressly illegal. I will argue that the citizenship test as its intended use in the resolution is not the same as a literacy test, and also that literacy tests are not inherently unconstitutional 

.
.
.
.
.

2A) Having voters pass A Citizenship/Naturalization Test is not the equivalent of having voters pass a Literacy Test

A literacy test, by definition and in the context of US political history, is a test that evaluates nothing more than a persons ability to read or write:

A citizenship/naturalization test given to potential immigrants hoping to achieve citizenship in the United States is broken into two parts, a civics test that evaluates knowledge of US history and how the government works, and an english test that examines a person's ability to read, write, and speak.


On the surface, one could mistakenly believe in this debate that a citizenship test and a literacy test are effectively one in the same, but thats not really the case. I could also see how the wording of the resolution of the debate could give that impression, so allow me to clarify. 

In THIS context of the resolution being debated, I have only ever argued that the test given to potential voters to evaluate their knowledge of "basic facts about the world", and would be about "testing a persons knowledge about American affairs to a good degree".... I even list several examples of what the test given to voters should be about, which would be "Questions [about] who is the current VP, Why does the flag have 50 stars, Presidential terms last how many years, etc"

Nowhere in my opening arguments did I ever argue that voters should be required to pass a citizenship test to demonstrate they can read and write in good english, nor do I argue that the quality of people elected to powerful positions would be improved if voters were screened based on their ability to read or write...... The main argument of the entire resolution is that voters should have to pass a test of basic knowledge and facts in order to be able to vote for presidents and senators. I then use the citizenship test given to immigrants seeking to become citizens as the basis for what that test would be, since it would test knowledge of important basic facts and also be fair and unbiased. 

Con's argument that a citizenship test for voters in this context amounts to a literacy test is fairly weak, as evidenced by the fact that earlier in his own arguments, Con concedes that they are not alike at all:

"The U.S. Citizenship is not a literacy test (at all) and doesn't test linguistic capabilities beyond studying a set of predetermined questions" - Con in Round 2. Argument about 15th Amendment.
Con claimed early on in his arguments that a citizenship test is not a literacy test at all, only to claim otherwise in the second half of his arguments because he needed something for his arguments to stand on......

If Con decides to flip flop on his previous claim and make an excuse that he simply misspoke the first time, thats fine. People make mistakes..... If Con accuses me of trying to move the goalposts of the debate, thats admittedly fair, I could have provided better clarity for what exactly Im debating in favor of with this debate in the opening rounds..... Either way, this would then bring us to the other aspect of why his argument against the resolution (that it amounts to a literacy test and is therefore illegal) fails: which is the fact that literacy tests are not inherently unconstitutional.

.
.
.
.
.

2B) Literacy Tests are not inherently Unconstitutional. 

Literacy tests were outlawed as part of an add-on to the Voting Rights Act in 1970, which itself was made to shore up loopholes of the 15th Amendment that granted African American males the right to vote. Following the end of the Civil War, concepts and devices such as poll taxes, literacy tests, and lack of redistricting were used by conservative southern states to suppress the voting rights of African Americans. The suppression of voting rights of African Americans became an issue in the 1960's, and rather then opt to have literacy tests be enforced onto everyone equally and across all states, Congress opted to do the 'easier' and 'safer' thing by just banning the practice nationwide. 

Literacy tests were only banned because southern states used it to violate the voting rights of African Americans. If southern states had applied literacy tests to everyone equally, they would not have been viewed as a tool of biased voter suppression, and would not have been banned by the Voting Rights Act and its subsequent add-ons. 

That may be a bold claim to make to some people who are inherently skeptical. Im sure that if I left it at that and tried to move on, Con would be more than eager to challenge that assumption and argue that even if literacy tests were applied equally, they still would have been found unconstitutional. 

It may come to a surprise then that Supreme Court has already ruled that literacy tests applied equally to everyone are not unconstitutional

In Lassiter v. Northampton County Board of Elections, (1959), the U.S. Supreme Court held that literacy tests were not necessarily violations of Equal Protection Clause of the Fourteenth Amendment nor of the Fifteenth Amendment

Several years before the Voting Rights Act was even drafted, the Supreme Court themselves ruled in a case that literacy tests weren't necessarily unconstitutional. Not only were they not inherently unconstitutional, the Head Justice for the Supreme Court went an extra step further, stating that literacy tests were perfectly legal/Constitutional if applied they are applied to everyone:

"The opinion of the court, delivered by Justice William O. Douglas, held that provided the tests were applied equally to all races, were not "merely a device to make racial discrimination easy," and did not "contravene any restriction that Congress, acting pursuant to its constitutional powers, has imposed," the literacy test could be an allowable use of the state's power to "determine the conditions under which the right of suffrage may be exercised."
In other words, not only has the Supreme Court themselves stated that literacy tests are not inherently unconstitutional, the Supreme Court also stated that literacy tests given to everyone equally and not administered in a discriminatory factor is perfectly constitutional. 

Therefore, the argument that requiring voters to pass a citizenship test to be able to vote for presidents and senators is illegal is simply wrong..... The resolution of the debate was intended to focus more on testing the civics knowledge of voters rather than english proficiency in voters for the betterment of the country, and the Supreme Court themselves have ruled that the use of literacy tests applied equally to everyone is not unconstitutional..... While the current Voting Rights Act prevents individual states from making their own literacy tests today, Congress only needs to decide to pass a law saying otherwise in order for it to be legal again. If we go one step further and have all voters across the nation be required to pass a citizenship test to be able to vote for senators or presidents, as is being argued in the resolution of this debate, such a proposal would not be unconstitutional, based on evidence from the Supreme Court themselves. 

(On a side note, the constitutionality of literacy tests also related back to argument 1B that argues that US voting beliefs go beyond just being able to give consent and be a citizen. If the Supreme Court rules that literacy tests are legal if applied to everyone equally, it goes in direct violation of cons claims that voting is only about ability to consent and having citizenship, otherwise literacy tests would be considered unconstitutional regardless of how it is applied)

.
.
.
.
.

2C) Passing a citizenship test is a pretty decent measure of deciding whether or not someone knows enough about the world and about the US government to be able to vote for good reasons

The last portion of Con's argument against the resolution is that because the US citizenship test does not test for common facts like the ones I mentioned in round 2 (Does the Earth revolve around the Sun, What is Auschwitz, etc) it would therefore do nothing in terms of signaling which voters are smarter or more intelligent. 

This argument is both blatantly false and a bit of a mischaracterization of what the overall goal of the resolution is. Having voters be able to pass a citizenship test in order to vote for president or senators will not eliminate ALL voters who could be ignorant about important issues..... Thats not what the goal of the resolution is.... The goal of having voters be required to pass a citizenship test to be able to vote for senators and presidents is that it would REDUCE the number of uninformed voters casting uninformed votes that could alter the outcome of elections for the worse, as I mentioned in this quote from the conclusion of my 1A argument: 

"If (uninformed) voters are weeded out by failing to pass a citizenship test, then the influence that these people would have in electing politicians would be substantially reduced"
Furthermore, voters who think the sun revolves around the Earth or don't know ANY branches of government would almost certainly not know other questions that are on the citizenship test. A study that asked Americans questions that are found on citizenship tests found that 16% of Americans missed 4 out of 5 randomly selected questions, meaning they only got 1 question right at most out of 5 https://www.usatoday.com/story/news/politics/elections/2016/06/30/poll-americans-citizenship-test/86559188/ . I'm willing to bet that a good chunk of that 16% of Americans are also among the 33% of Americans who cannot name ONE branch of the government, or the 60% of millennials who don't know what Auschwitz is. 
Con
#6
In Round 2 I was referring to Round 2 as 'Round 1' and Round 3 as 'Round 2' this was a mistake as I was writing it mentally thinking that since it was the first round of debate that Round 1 had not come and gone yet.

Also, full credit to https://www.law.cornell.edu/constitution/amendmentxvii for my 17th Amendment quote.

S&G: I am using British English outside of quotes.

There exists three reasons that Pro and I disagree and one significant thing we both completely agree on in this debate.

The thing we agree on is:

If the US were to enact a forced US Naturalisation/Citizenship Test for voters to pass in order to vote, it would be most certainly best applied to the Senate and Presidential votes as these demand the most from voters and impact the nation as a whole.

Where we disagree are the three following things:

1) The test, if any, should be the US Citizenship Test.
2) That it flies in the face of Constitutional Amendments and Legal adjustments over time that have strongly implied that voting is supposed to be accessible to all who can decide for themselves and are citizens of the US (by location of birth or by parental nationality as those two don't need to take the test to become a US citizen).
3) That testing for knowledge is going to lead to us having more intelligent leaders which is in any way going to filter out disloyal voters and leaders.

I am going to go point-by-point of what Prop brings up in R3 and make it clear why either they are wrong or I am right and/or rather why we are both right and why the resolution is actually nothing to do with the point if it's appearing to be against it.

When you think of 'American ideals' and other 'core' things that the US likes to claim it stands for/is the protector of, a few things can come to mind.... "Defending Democracies overseas", "Defending Human Rights", "Freedom from Tyranny", "Freedom of Speech and Religion", "Right to Defend oneself" are all ones I can think of off the top of my head. You can probably think of many more that I haven'e even listed. 
- Prop
I do not know why defending democracies overseas is an inherently US value (and is something many Libertarians[1] and Progressives[2] are against) or where Pro is drawing these from but what I do know, for a fact is that if we admit that democracy is a US core value that the best way to achieve democracy is to allow as many who are proven capable of deciding for themselves (capable of actual legal consent) then we will get closer to determining who most consent to being leader. This is the simplest concept on which the entire US value of democracy is based and has been proven to be where the laws are moving to over time as I proved in Round 2.

How will you get freedom from Tyranny if they start filtering out people who believe non-conventional beliefs or are perhaps not supportive of NASA because they are curious about why the organisation is never audited since it's entitled to very high level security clearance? There's so much to go into in Pro's case that makes it crystal clear why the very values they have stated here are defeated by the policy that the resolution wants enacted.

All of these things can be jeopardized when uninformed voters are able to sway the outcome of a close contest to elect worse politicians the nation would otherwise get.
- Prop
Alright, but who are you to say who is 'worse'? What is going to jeopardise democracy more than anything else is a way to restrict voters who are concluded to be full-fledged, loyal citizens of the US and who are banned from voting due to a lack of knowledge or who oppose some mainstream ideas. 
A voter who thinks the sun orbits around the Earth could conceivably believe that global warming is a lie as well, and therefore vote for a politician who wants to drastically expand government surveillance laws, simply because that politician said in a commercial that he also doesn't believe in global warming.
There is plenty of reason to believe in flat Earth and even if you consider it unscientific to be suspicious of NASA which never gets audited except when it is noticed that it's wasting billions[3] or to be suspicious if we ever went to the moon which has a plethora of evidence behind it that is simply not approved by the overpowering influence of the government-supported NASA somehow going to make you a worse voter? In fact which side between round earthers and flat earthers is the bullying side based upon blind faith?

"Hopefully the truth will come out."

"Woodrow Wilson was the US president in the 1920s and he said there was a shadow government above him pulling the strings."

"I am going to put it all behind me. I've not enjoyed this process in the slightest."

"I got kicked out of church, I got unfriended by many people because of what I believe, that the Moon landings are fake."
- [4]

Oh and do you know which actually came first? It's often said that we all used to believe the Earth was flat and bully round earthers but it's far from that simple or that order of which came first:
Contrary to popular belief, it’s a misconception that many societies of serious, educated people ever actually believed in the flat Earth theory. “With extraordinary few exceptions, no educated person in the history of Western Civilization from the third century B.C. onward believed that the Earth was flat,” historian Jeffrey Burton Russell noted in 1997. “A round Earth appears at least as early as the sixth century B.C. with Pythagoras, who was followed by Aristotle, Euclid, and Aristarchus, among others in observing that the earth was a sphere.”
- [5]

Not to mention, why is it even necessary to prove an outlier correct? Why is it necessary for everyone to be identically informed and to know exactly the same as one another in order to permit them to vote? It's easy to say the following:
 A voter who doesn't know what the 1st Amendment is could vote for a politician who wants to ban all mosques just because the politician talks the same way he does. A voter who only listens to Alex Jones for news may vote for a politician that would outlaw gay marriage just because that candidate is a man and the other he is facing is a woman. 
- Prop

But why can't a voter be against freedom of speech? Why can't voters support amending it and completely removing from the US the philosophy that saying whatever you want is alright? Alternatively, why should voters who don't know the Amendment but who support it be banned from voting? Yes, that's right the US Citizenship Test method of screening would:

Prevent people who actually support the First Amendment's core right/value (freedom of expression) and don't know what exactly the Amendment is on a test to score too low to vote but would allow a voter who opposed it and knew that it was the First Amendment to vote.

Not to mention, the Alex Jones part is most hilarious considering who is oppressing who in recent times.[6][7] Furthermore, it is just as outrageously undemocratic to ban the ability of those who are anti gay marriage to vote and against frankly anything you hold sacred to vote as to ban those who support it to vote.

Let's be crystal clear here:

Pro has no more right than those he opposes to prevent someone believing in his core values or theirs to vote.

A concept so sacred to voting in the US is that everyone gets to vote. I proved that already in Round 2. The reason why the youth can't vote is similar to why Prisoners can't. Oh yes, that's right it's the same reasoning. Even an ex-con can be argued to have displayed an insufficient capacity to control themselves and properly consent to things which is why they insufficiently obeyed the law, or so is the logic. Also, Pro has to explain why we should keep the law banning ex-cons from voting since clearly an intelligent and well-informed ex-con is going to have more right to vote in the eyes of Pro than a less informed law abiding citizen (which completely runs against how the current system works).

From sex to the ability to justify putting someone in a cage against their will, the entire concept of prison and laws regarding it show that breakers of the law are held to a lower level of consent than other adults:

The Prison Rape Elimination Act (PREA) was passed in 2003 with unanimous support from both parties in Congress. The purpose of the act was to “provide for the analysis of the incidence and effects of prison rape in Federal, State, and local institutions and to provide information, resources, recommendations and funding to protect individuals from prison rape.” (Prison Rape Elimination Act, 2003). In addition to creating a mandate for significant research from the Bureau of Justice Statistics and through the National Institute of Justice, funding through the Bureau of Justice Assistance and the National Institute of Corrections supported major efforts in many state correctional, juvenile detention, community corrections, and jail systems.

The act also created the National Prison Rape Elimination Commission and charged it with developing draft standards for the elimination of prison rape. Those standards were published in June 2009, and were turned over to the Department of Justice for review and passage as a final rule. That final rule became effective August 20, 2012.

In 2010, the Bureau of Justice Assistance funded the National PREA Resource Center to continue to provide federally funded training and technical assistance to states and localities, as well as to serve as a single-stop resource for leading research and tools for all those in the field working to come into compliance with the federal standards.
- [8]

Also, the concept of felons/prisoners voting is down the the State (especially ex-cons). The reason why prisoners and ex-cons can lose the right to vote is often to do with displays of complete disloyalty to the US and disregard for the US's current law as a whole. Again, the way we would determine this is clearly by actions and not by a knowledge-based examination/test.

If a person is convicted of first-degree murder in the state of Vermont, he or she will retain the right to vote — even while incarcerated.

But a person who commits perjury in Mississippi could be permanently barred from casting a ballot there.

It is up to states — not the federal government — to say whether convicted felons can vote, and which ones, and when. So the rules for convicted criminals can change, sometimes drastically, from one state to the next.
- [9]
In most states, felons cannot vote while they are in prison but can regain their voting rights after they are released (as in Massachusetts and Hawaii), after they complete their parole (as in Colorado and Connecticut), or when they are no longer on parole or probation (as in New Jersey and Texas).

California relaxed its rules a little in 2016. Convicted felons sentenced to county jails there can now vote while in custody, but the shift did not apply to those who were sentenced to a state or federal prison.

And there are two states that do not revoke criminals’ right to cast a ballot: Vermont and Maine. There, felons can vote even when they are behind bars.

“The state disparities are really astounding,” said Christopher Uggen, a professor of sociology and law at the University of Minnesota who also worked on the 2016 Sentencing Project study. “It is definitely confusing at election time, and many former felons are risk-averse — they may not vote if they are afraid of getting a felony conviction for illegal voting.”
- [9]

The concept of voter ID is not only under a lot of heat and opposition[10][11] but frankly have nothing to do with screening out those who 'don't know something' in any shape or form.
Voter identification laws are a part of an ongoing strategy to roll back decades of progress on voting rights.  Thirty-four states have identification requirements at the polls. Seven states have strict photo ID laws, under which voters must present one of a limited set of forms of government-issued photo ID in order to cast a regular ballot – no exceptions.

Voter ID laws deprive many voters of their right to vote, reduce participation, and stand in direct opposition to our country’s trend of including more Americans in the democratic process. Many Americans do not have one of the forms of identification states acceptable for voting. These voters are disproportionately low-income, racial and ethnic minorities, the elderly, and people with disabilities.  Such voters more frequently have difficulty obtaining ID, because they cannot afford or cannot obtain the underlying documents that are a prerequisite to obtaining government-issued photo ID card.
- [10] 
After the 2013 Shelby County v. Holder Supreme Court case weakened federal oversight over state and county election laws, the debate over whether these and other more restrictive laws have discriminatory effects has mostly been waged in the realms of ideology and intent, with most existing studies relying on data limited by time, place, or bias.

The catch-22 of course is that the laws have to be passed and solidly in place first to have robust longitudinal data on their effects, which in this case would mean potentially discriminatory effects would have already impacted elections. A new study from researchers Zoltan Hajnal, Nazita Lajevardi, and Lindsay Nielson at the University of California San Diego is one of the first to analyze certified votes across all states after the implementation of voter laws in multiple elections, and it found just that kind of racially discriminatory impact.

Specifically, they found “that strict photo identification laws have a differentially negative impact on the turnout of Hispanics, Blacks, and mixed-race Americans in primaries and general elections.”

The authors note that the existing research tends to point to three things: that strict voter ID laws requiring identification to cast a ballot do in fact reduce turnout by some amount, that turnout reduction tends to work in Republicans’ favor, and that differential effects have been observed along class and education lines, but not race. But the UCSD researchers call those conclusions into question, noting that analyses based on elections data before 2014 could not have collected comprehensive enough data to rule out racial suppression, and that analyses that sidestep that limitation by relying on survey data tend to fall victim to people of color over-reporting if they voted in prior elections.
- [11]

In fact the entirety of Pro's case is filled with so many reasons to oppose the resolution. The US Naturalisation/Citizenship Test is not going to filter out flat-earthers at all because people who believe the Earth is flat are more likely than Round-Earthers to know precisely what NASA is, which department of the government funds it and the details of what it has brought forth regarding Global Warming. That's right, contrary to popular belief, the flat earthers are the ones who are likely to know those details and pass the space-funding section of the test regarding what tax money goes to fund more than your average round earther since they are so concerned with that money as they consider it wasted and NASA to be enacting an elaborate lie. Extremely similarly, people who are actually against freedom of speech and other means of expression are going to know exactly what the First Amendment is since they will be fighting against it and know what their opposed legislation is whereas it's far more likely that someone who is happy to have that freedom and takes it for granted isn't going to know too specifically which Amendment it is and is likely to be the one to fail that part of the test.

Let's ignore both those factors. If a flat-earth supporting, global-warming denying and/or free-speech opposing candidate is able to run for President and gain an active voter base that would mean it is the Media and correctly-informed candidates and news stations who should put in the work and improve their capability to inform voters. It, in no way at all, is the voters who should be restricted and punished. The minute we begin to punish voters for knowing too little who are full fledged citizens of the US, we are not taking responsibility for whose fault it is that they are uninformed and not going to have candidates promising to better educate and inform the populace since that would cost more money and the informed wouldn't care about the oppressed uninformed knowing more (and over time they will know less and less so on and so forth).

On top of this, the US Naturalisation/Citizenship Test is a bit of a joke when it comes to legality. Not only is half of it a literature test which is outright illegal due to the 1970 and 1975 Amendments but most of the Civics Test is going to do nothing but punish voters for being poorly taught in schools and by the media things that they should be regularly told and informed of.

I believe I have fully made my case here and sufficiently torn apart Prop's Case at its fundamentals let along its added points on top.

Also, the fact that Prop keeps referring to other ways that voters are restricted in order to justify this one is in no way at all relevant to the debate since I have already explained that the current restrictions are nothing to do with knowledge and that they themselves (especially voter IDs) are being opposed heavily from many angles.

You cannot test people for knowledge, justify it as wanting intelligent leaders and then say you have filtered out disloyal or sinister voters and leaders at all. I hope I have made this crystal clear.

Sources (All Accessed last 29th August 2018)
Round 4
Pro
#7
Because this is the final round of the debate, I will refrain from introducing any new arguments and try to keep my own remarks short. Due to how the focus of the arguments on both sides have ping-ponged a couple different times in the 2nd and 3rd round, Con is welcome to make final rebuttals of his own in his final round rather then stick to only conclusions 

=============================================================================================

List of arguments Con has forfeited from round 3: 

1) Having voters pass a Citizenship/Naturalization Test is not the equivalent of having voters pass a Literacy Test

2) Literacy Tests are not inherently Unconstitutional, and The Supreme Court has found that states who use literacy tests correctly can be perfectly Constitutional

By conceding the majority of arguments that have been introduced so far in round 2 and now round 3, Con's main defense against the resolution has been reduced to just two main 'arguments'.

The first is the argument that having voters pass a citizenship test in order to vote for senators or presidents is bad/un-American. The second is the argument that having citizens pass a citizenship test in order to vote for senators and presidents will not be effective in electing better leaders, and somehow have the opposite effect by leading to tyranny..... A sizable chunk of con's responses are also built on straw mans and misunderstandings about what the debate resolution is proposing in the first place, which will be addressed together in separate sections. 

.
.
.
.
.

1) It is not against US values to have citizens be able to pass a citizenship test in order to be able to vote for presidents or senators. 

"If we admit that democracy is a US core value, than the best way to achieve democracy is to allow as many who are proven capable of deciding for themselves (capable of actual legal consent)" 
As mentioned before in the previous rounds, that is just Con's opinion. There are many laws on the state and national level that restrict voting beyond just being able to give consent and be a citizen. These laws include Voter ID laws.... Voter registration laws.... Loss of ability to vote if you've committed a felony..... And loss of ability to vote if you don't vote often just to name a few.

Many of these laws have long been and even recently been upheld as Constitutional by the Supreme Court itself (see next quote). Since they are the most qualified court in the entire world to decide the meaning and intent of the Constitution (which is where core American values are drawn from) then Cons assertion of what US democracy is 'really about' is misguided and different from what it is 'actually' about, according to those most certified (The Supreme Court) to define that is. 

"In a decision by Justice Samuel Alito, the court emphasized that [...] not only “are States allowed to remove registrants who satisfy these requirements, but federal law makes this removal mandatory

Supreme Court Majority Opinion on the case ruling that Ohio's voting registration purge was constitutional, and that a person who did not vote in a previous election could have their registration stripped: http://www.scotusblog.com/2018/06/opinion-analysis-justices-rule-for-ohio-in-voter-registration-dispute/
If the Supreme Court rules that states have not only the right, but the REQUIREMENT, to remove voters from registration lists if they do not participate in voting regularly, it completely invalidates Con's claim that US democracy is about "allowing as many who are proven capable of deciding for themselves" to be able to vote, since frequency/infrequency of voting has nothing to do about ability to decide for themselves, yet has been found required and Constitutional according to the Supreme Court. 

Furthermore, as mentioned in the previous round in an argument that Con blatantly forfeited, the Supreme Court has also found that states being able to test voters based on their literacy is perfectly Constitutional, as long as the state applies the test to all voters equally, rather than just specific parts of society such as minorities: 

"In Lassiter v. Northampton County Board of Elections, (1959), the U.S. Supreme Court held that literacy tests were not necessarily violations of Equal Protection Clause of the Fourteenth Amendment nor of the Fifteenth Amendment. "

"The opinion of the court, delivered by Justice William O. Douglas, held that provided the tests were applied equally to all races, were not "merely a device to make racial discrimination easy," and did not "contravene any restriction that Congress, acting pursuant to its constitutional powers, has imposed," the literacy test could be an allowable use of the state's power to "determine the conditions under which the right of suffrage may be exercised."

In summary: If the Supreme Court has found it constitutional to bar voting from people who cannot pass a literacy test, and has found it constitutional for states to revoke voting rights from those who do not do it actively, it completely tears apart the idea that the US core value of democracy is "allowing as many people to vote as possible who are proven capable of making decisions for themselves", because clearly there is more to democracy then that....... If Pro disagrees with the opinions of the Supreme Court for what democracy in the US should be about, then it doesn't matter, because the Supreme Court is the ultimate authority on what the Constitution stands for, not Con. 

.
.
.
.
.

2) Screening out voters who cannot pass a citizenship test, and would be unable to vote for senators or presidents, would not lead to tyranny

How will you get freedom from Tyranny if they start filtering out people who believe non-conventional beliefs?
First off, It is not a 'non-conventional belief' to not know what the 1st Amendment is, nor is it a 'non-conventional belief' to not know what Auschwitz is. Con completely mistakes non-conventional beliefs with pure ignorance, and its pure ignorance that helps lead a society from freedom to tyranny.

Second, the resolution only filters out people from voting for presidents and senators. It does not bar people who do not pass the test from voting at all, leaving other avenues and places for their voices to be heard and respected (House of Representatives + State Governors)

Third and most importantly, people with a great number of non-conventional beliefs will still be perfectly capable to pass a citizenship test, because a citizenship only tests basic knowledge of the US government such as how many stripes are on the flag or who the current Vice President is..... It does not explicitly target people who think the earth is flat or think vaccines are bad. Instead, the test screens out the very lowest and most unknowledgeable voters based on how much they don't know basic established facts about the United States itself, not based on advanced scientific theories or intricate economic philosophies.  

What is going to jeopardise democracy more than anything else is a way to restrict voters who are concluded to be full-fledged, loyal citizens of the US and who are banned from voting due to a lack of knowledge or who oppose some mainstream ideas. 
First off, they would not be banned from voting outright. People who fail to pass a citizenship test would still be able to vote in a vast majority of elections that those who do pass a citizenship test would also be able to vote in. The only elections they would not be able to vote in would be senatorial and presidential contests, which is what the resolution of the debate is about in the first place.  

Secondly, Democracy is jeopardized the most when its fate can lie in the hands of a voter base that has been lied to by leaders, is ignorant of how the government even works, is misinformed about important affairs the nation has a prominent role in, or even hateful and spiteful towards another branch of society/minority..... While having voters be required to pass a citizenship test wouldn't eliminate all of those who are alarmingly ignorant about things they should know about, it would still be an improvement in reducing the potential impact that a small branch of society could have in plunging the rest of the country itself (and possibly the world by extension) into tyranny. 

The US Citizenship Test method of screening would prevent people who actually support the First Amendment and don't know what the Amendment is on a test to score too low to vote, but would allow a voter who opposed it and knew that it was the First Amendment to vote.
Requiring voters to pass a citizenship test in order to vote for senators and presidents would just as equally prevent people who are against the First Amendment and don't know what it actually covers from being able to vote for senators and presidents as well. In other words, the resolution would not only screen out those who blindly support something they don't know anything about, it would also screen out those who blindly oppose something they don't know anything about.

Con may try to argue that every person who is against something (such as the 1st Amendment) would have to know about it, but thats not necessarily the case..... Americans have been shown to be opposed to many different things they don't actually know important information about, even from the most recent past, as evidenced in the following links:

- In 2017, studies found that over 80% of people opposed the repeal of Net Neutrality legislation passed in 2017, yet 72% of Americans are unsure of what 'Net Neutrality' even MEANS, let alone what was actually in the legislation. 


- In 2016, 64% of people opposed the GOP tax bill that was eventually signed into law by Trump, yet half of Americans didn't even know that the income tax brackets were changing, and half of Americans don't even know what their OWN tax bracket is, 


- In 2017, 63% of Americans were against moving the US embassy in Israel from Tel-Aviv to Jerusalem, yet 75% of Americans couldn't even find Israel on a map in an earlier study. 


- 30% of Americans are opposed to LGBT marriage rights, but almost half of Americans don't even know what LGBT stands for. (The higher awareness of millennials for what LGBT means and their support for LGBT rights indicates those who opposed it were the ones more likely to not know what it stands for)


The point is, people who oppose something do not necessarily know everything about the thing they are opposing, and may not know anything at all about the thing they are opposing.... The claim that the resolution would screen out people in favor of the First Amendment and don't know anything about it, but would not screen people against the first amendment who do know about it, is misguided.... Because Americans have a tendency to (sometimes vocally) be against something that they do not actually know anything about.

In this case, and in just about every case, there are people on both sides of an issue that will not know anything about the issue, and will not know basic information about the US itself. These people would be screened out equally based on their lack of basic knowledge of how the US government works or basic facts about the US itself. Any fear that this system could lead to some tyrannical faction being able to rise up and strip away US rights is just conspiracy theory, because it operates under the assumption that everyone who supports the tyrannical side will know about what it is they are trying to eliminate, and previous links indicate that its not the case that the opposition always knows what it is they are opposing. 

.
.
.
.
.

3) Other straw mans/conspiracy theories from Con's arguments 

A great amount of con's previous arguments are not actually grounded on the actual arguments I presented at the start of round 3 (he forfeited a majority of those). Instead Con either completely misunderstands or intentionally warps what the resolution of the debate is even about. These straw mans that he sprinkles throughout his arguments are responded to here, so as to keep the few parts of the debate currently not bogged down in con's conspiracy theories from being lost in massive amounts of text. 

Pro has to explain why we should keep the law banning ex-cons from voting since clearly an intelligent and well-informed ex-con is going to have more right to vote in the eyes of Pro than a less informed law abiding citizen
I never said nor anywhere implied that a person who is able to pass a citizenship test is "guaranteed" the right to vote as Con claims. A person who is able to pass a citizenship test could still rightfully be barred from voting for other reasons such as not having citizenship, not being registered to vote, or being a felon. This resolution does not override all other laws regarding who is able to vote where anyone who passes the test can vote regardless of criminal history or citizenship. Instead, this resolution would add to the list of other limits on voting (ID laws, registration, criminal history, age, mental sanity) the qualification of 'having basic knowledge of the US'. 

Why is it necessary for everyone to be identically informed and to know exactly the same as one another in order to permit them to vote?
The point of having voters pass a citizenship test in order to vote for presidents and senators is not even close to being the same as "having everyone be identically informed and know exactly everything that everyone else does".... The civics portion of citizenship tests are not all-encompassing tests of a persons knowledge that relate to every important thing worth considering when supporting a candidate for office.... Instead, citizenship tests evaluate a persons knowledge of BASIC information and facts about how the US government works, such as how many branches of government they are and what is a right covered in the 1st Amendment. 

Pro has no more right than those he opposes to prevent someone believing in his core values or theirs to vote.
It's not a core value to not know what the First Amendment is, or to not know who the current Vice President is. That you think otherwise is incredibly concerning..... As stated before, citizenship tests to not test people on core values or general beliefs they hold. It tests understanding of incredibly basic facts about the government that the average middle schooler has a 50-50 shot at being able to guess right.

People with completely different core values could still be able to vote for senators or presidents by passing a citizenship test, because citizenship tests only test basic facts about the US government, not scientific theories or personal philosophies. 

Voter ID [...] frankly has nothing to do with screening out those who 'don't know something' in any shape or form.
A voter being able to pass a citizenship test would not supersede being unable to produce acceptable forms of identification. The current resolution would be implemented concurrently with already existing restrictions towards voting such as ID laws and criminal histories, it does not give those without ID a free pass to vote like you imply.  

"The fact that Pro keeps referring to other ways that voters are restricted in order to justify this one is in no way at all relevant to the debate since I have already explained that the current restrictions are nothing to do with knowledge and are being opposed heavily from many angles.
They don't have anything to do with knowledge but, as mentioned repeatedly now, they prove that your claim the core belief of US democracy, "As many people being allowed to vote as possible as long as they can make a decision and are citizens" is false, since criminal history, ability to provide identification, and previous voting activity/inactivity have all been upheld by the Supreme Court as Constitutionally valid restrictions on voting......

Therefore, they are perfectly relevant to the debate, because they prove the last argument you have not built on straw mans or misinterpretations of the resolution to be baseless and incorrect. 

On top of this, the US Naturalisation/Citizenship Test is a bit of a joke when it comes to legality. Not only is half of it a literature test which is outright illegal due to the 1970 and 1975 Amendments....
Literacy tests are only illegal because Congress made it illegal since it was the easy thing to do back in the 1970's when conservative southern states were using it as a tool of discrimination against black people. If Congress decided that literacy tests were ok, as the Supreme Court has decided, and decided to pass the resolution of this debate, it would become legal. Congress only needs to pass a law saying that literacy tests are legal in order for it to be legal again, since Congress was the one that said it would be illegal in the first place, and Congress has the power to undo past legislation of its own. 

.
.
.
.
.

4) Unsupported opinions from Con

"If a flat-earth supporting, global-warming denying and/or free-speech opposing candidate is able to run for President and gain an active voter base that would mean it is the Media and correctly-informed candidates and news stations who should put in the work and improve their capability to inform voters"
Opinion. 

 "It, in no way at all, is the voters who should be restricted and punished"
Opinion that is also disputed by the Supreme Court. The Supreme Court has ruled in the past that a state has the implied power to screen out voters from being able to vote if they fail to pass a literacy test, as long as the state applies the literacy test to all voters equally, such as how this debate resolution would. 


"The minute we begin to punish voters for knowing too little who are full fledged citizens of the US, we are not taking responsibility for whose fault it is that they are uninformed"
Opinion. How many voters willingly choose to live in their own little bubble where they refuse to acknowledge things that go against their own views? The prevalence of social media sites have allowed for voters to construct 'digital echo chambers' for themselves to never hear any information they don't want to hear. It is not always the fault of society or the educational system as a whole if a person is ignorant because they choose to only view news and sources that reinforce their ignorance.

https://www.npr.org/sections/alltechconsidered/2016/07/24/486941582/the-reason-your-feed-became-an-echo-chamber-and-what-to-do-about-it
https://www.ted.com/talks/theo_e_j_wilson_a_black_man_goes_undercover_in_the_alt_right/transcript?language=en

.
.
.
.
.
.
.
.
.
.
.
.
.
.
.
.
.
.
.
.
.
.
.
.
.
.
.
.
.
.
.
.
.
.
.
.


To summarize this debate and give a quick cliff-notes version of what has happened to voters who's eyes are getting sore:

List of Arguments Con has forfeited:

1) Con forfeits the argument that the US is too powerful and important in world events to have to deal with bad leaders for an extended period of time (Argument 1B from Round 2) 

2) Con forfeits the argument that the US President and Senators have an incredible ability to impact world events and US policy (Argument 2A from Round 2) 

3) Con forfeits the argument that US Representatives, State Governors, and County level politicians have almost no impact on national legislation and have almost no voice outside of their own state (Arguments 2B, 2C, 2D, and 2E from Round 2) 

4) Con forfeits the argument that ability to pass a literacy test should only be applied to Presidential and Senate elections (The conclusion of Arguments 2A to 2E from round 2) 

5) Con forfeits the argument that restricting voting rights of uninformed voters enables the US government to better pursue/defend its other core ideals and beliefs (Argument 1A from Round 3, Con instead argues that restricting voting rights leads to tyranny) 

.
.
.

List of Arguments Con has not acknowledged or misunderstood:

6) Con ignores the argument that both by definition and by his own previous admission that a naturalization/citizenship is not a literacy test "at all" (Argument 2A from round 3) 

7) Con ignores the argument that literacy tests are not unconstitutional, and that if applied equally to all voters, literacy tests are perfectly constitutional according to the Supreme Court (Argument 2B from round 3) 

8) Con misunderstands the resolution that voters who fail to pass a citizenship test would only be barred from voting for presidential and senatorial candidates (Argument 2 from this round) 

9) Con misunderstands the resolution that voters who do pass a citizenship test could still be barred from voting due to not being registered voters or for having a criminal history (Argument 2 from this round)

.
.
.

List of arguments by Con that have failed

10) Con loses the argument that the US's 'core' belief about Democracy is having allowing as many people to vote as possible if they can make decisions for themselves and are citizens, because many US laws restrict ability to vote beyond just being able to give consent and be a citizen, which are laws that have long been and recently upheld by the US Supreme Court as Constitutional

11) Con loses the argument that restricting US voting rights will 'lead to tyranny' by giving one tyrannical side an advantage over another pro-democracy side, because evidence shows that even people who are opposed to something often do not know what it is they are opposing in the first place, meaning that both sides of any issue will be impacted by the resolution, since ignorance of important details is never limited to just one side of a debate. 


Based on Con's forfeiture, misunderstanding, intentional ignoring, and failure to dispute these 11 arguments, the resolution should be affirmed that US voters should be required to pass a citizenship test in order to vote for presidents and senators. (1) The resolution would screen out exceptionally ignorant people from being able to influence an election for the worse in elections that hold great significance, (2) using a citizenship test given to immigrants would be an unbiased way to test knowledge, (3) it would allow the US to elect better leaders to help make better decisions and defend its core ideals, (4) the test would only evaluate a persons knowledge of how the US government works and not be biased towards any ideology or controversial belief, (5) the resolution is perfectly constitutional as evidence from Supreme Court rulings on voter restrictions and literacy tests suggests, and (6) it would not have the negative side-effects that con's conspiracy theories suggest it would since ignorance is found on both sides of any issue, not just one side of a debate that may go against democratic values. 

I encourage argument points to be awarded to me, while conduct, source points, and spelling/grammar be left tied. 

Thanks for reading!

.
.
.
.
.
.
.
.
.
.
.
.
.
.
.
.
.
.
.
.
.
.


Con
#8
I think in a debate of this depth it can be hard for voters, by the end, to backtrack exactly what has gone on. It's up to me to inform you (the voters) of what's going on and not rely on your own intellect or knowledge in order to judge this debate well. The more I inform you, the better you vote. See how that works? This is not a new point, this is precisely what my entire Round 3's ending was dedicated to prove.

Throughout this debate Prop has been pushing forth a notion of there being too severe a prevalence of stupidity amongst voters. In order to prove this, Prop provided shaky evidence at every point that highlighted that there are people with outlandish ignorance of facts (or what Prop believes to be facts). From Sceptics of the Holocaust to Sceptics of NASA and the Earth being round, Prop has kept highlighting that there are people who lack information (which offends or disgusts Prop on a personal level it seems) and that they cannot therefore have the right to vote until they prove they believe what others believe to be true as a matter of fact. Even if this alone doesn't disgust or worry you, I fully combatted this line of attack in Round 3. I explained the following:

The US Naturalisation/Citizenship Test is not going to filter out flat-earthers at all because people who believe the Earth is flat are more likely than Round-Earthers to know precisely what NASA is, which department of the government funds it and the details of what it has brought forth regarding Global Warming. That's right, contrary to popular belief, the flat earthers are the ones who are likely to know those details and pass the space-funding section of the test regarding what tax money goes to fund more than your average round earther since they are so concerned with that money as they consider it wasted and NASA to be enacting an elaborate lie. Extremely similarly, people who are actually against freedom of speech and other means of expression are going to know exactly what the First Amendment is since they will be fighting against it and know what their opposed legislation is whereas it's far more likely that someone who is happy to have that freedom and takes it for granted isn't going to know too specifically which Amendment it is and is likely to be the one to fail that part of the test.
- Con R3

That covered most angles not ruling out the parts of the test that test their knowledge but I think the key part of this is my Round 2 remark:

Excerpt ... the other part of the Citizenship Test has absolutely nothing to do with intelligence, it tests for knowledge that sure, is partly to do with Politics but do not in any way imply the one who gets better percentage is a more intelligent voter.

Something I want to quickly establish here is that loyalty has nothing to do with intelligence, what Pro is actually advocating is to test for intelligence by testing for loyalty and then using a test that tests solely for knowledge.

Loyalty =/= Intelligence =/= Knowledge

I will leave this as it is for now and let Pro defend their case but I reiterate: Nearly every single thing that Pro said most voters don't know in Round 1 is not tested for in the US Citizenship/Naturalisation Test.

- Con R2

So tell me what exactly does Pro combat this with?

Let's take a look:

The last portion of Con's argument against the resolution is that because the US citizenship test does not test for common facts like the ones I mentioned in round 2 (Does the Earth revolve around the Sun, What is Auschwitz, etc) it would therefore do nothing in terms of signaling which voters are smarter or more intelligent.

This argument is both blatantly false and a bit of a mischaracterization of what the overall goal of the resolution is. Having voters be able to pass a citizenship test in order to vote for president or senators will not eliminate ALL voters who could be ignorant about important issues..... Thats not what the goal of the resolution is.... The goal of having voters be required to pass a citizenship test to be able to vote for senators and presidents is that it would REDUCE the number of uninformed voters casting uninformed votes that could alter the outcome of elections for the worse, as I mentioned in this quote from the conclusion of my 1A argument:

> "If (uninformed) voters are weeded out by failing to pass a citizenship test, then the influence that these people would have in electing politicians would be substantially reduced"
[Pro quoting themselves from R2]

Furthermore, voters who think the sun revolves around the Earth or don't know ANY branches of government would almost certainly not know other questions that are on the citizenship test. A study that asked Americans questions that are found on citizenship tests found that 16% of Americans missed 4 out of 5 randomly selected questions, meaning they only got 1 question right at most out of 5 https://www.usatoday.com/story/news/politics/elections/2016/06/30/poll-americans-citizenship-test/86559188/ . I'm willing to bet that a good chunk of that 16% of Americans are also among the 33% of Americans who cannot name ONE branch of the government, or the 60% of millennials who don't know what Auschwitz is.
- Pro R3

This is so full of logical fallacies that I wasn't sure where to begin in combatting it so what I did was indirectly address every single part of it in my Round 3 reply.

Prop says that he will filter out the stupid by testing for specific knowledge. I explained that not only do the conspiracy theorists or direct opposition to mainstream laws and institutions have far more likelihood to successfully pass that part of the tests than the mainstream believers do but furthermore that there is no link between knowledge and intelligence and additionally no link between either and loyalty to the US. Prop tries to combat this saying 'he is willing to bet' on something because one source on the Internet says that 16% of Americans missed 4 out of 5 randomly selected questions (which is not how you sample size a massive test since maybe the 4 they missed would have been the only 4 out of the whole exam). On top of that, Prop fails to explain why I am wrong in stating the following:

Let's ignore both those factors. If a flat-earth supporting, global-warming denying and/or free-speech opposing candidate is able to run for President and gain an active voter base that would mean it is the Media and correctly-informed candidates and news stations who should put in the work and improve their capability to inform voters. It, in no way at all, is the voters who should be restricted and punished. The minute we begin to punish voters for knowing too little who are full fledged citizens of the US, we are not taking responsibility for whose fault it is that they are uninformed and not going to have candidates promising to better educate and inform the populace since that would cost more money and the informed wouldn't care about the oppressed uninformed knowing more (and over time they will know less and less so on and so forth).

On top of this, the US Naturalisation/Citizenship Test is a bit of a joke when it comes to legality. Not only is half of it a literature test which is outright illegal due to the 1970 and 1975 Amendments but most of the Civics Test is going to do nothing but punish voters for being poorly taught in schools and by the media things that they should be regularly told and informed of.

I believe I have fully made my case here and sufficiently torn apart Prop's Case at its fundamentals let along its added points on top.

Also, the fact that Prop keeps referring to other ways that voters are restricted in order to justify this one is in no way at all relevant to the debate since I have already explained that the current restrictions are nothing to do with knowledge and that they themselves (especially voter IDs) are being opposed heavily from many angles.

You cannot test people for knowledge, justify it as wanting intelligent leaders and then say you have filtered out disloyal or sinister voters and leaders at all. I hope I have made this crystal clear.
- Con R3

Prop tries to call this all unsupported opinion but it's not. It is supported quite blatantly by the very part where I say:

The minute we begin to punish voters for knowing too little who are full fledged citizens of the US, we are not taking responsibility for whose fault it is that they are uninformed and not going to have candidates promising to better educate and inform the populace since that would cost more money and the informed wouldn't care about the oppressed uninformed knowing more (and over time they will know less and less so on and so forth).
- Con R3

That is literally the support of the opinion so it can not be called an unsupported opinion.

Do you know what can be called an unsupported opinion?

The following:
I will argue that the citizenship test as its intended use in the resolution is not the same as a literacy test, and also that literacy tests are not inherently unconstitutional.
- Prop R3

I will admit I made an error in saying that the US Citizenship test isn't a literacy test at all. What I meant to say was that it appears the test Prop is proposing is the US Naturalisation Test without the entire part of it (which is at least 40%) that tests for one's ability at the English Language. I apologise for my poor wording and concede the blatant fact that:
The English test has three components: reading, writing, and speaking. Your ability to speak English will be determined by a USCIS Officer during your eligibility interview on Form N-400, Application for Naturalization. For the reading portion, you must read one out of three sentences correctly. For the writing test, you must write one out of three sentences correctly.

Now let's remind ourselves just how unconstitutional a literacy test for voters is (and note that it went from being heavily endorsed to being universally opposed over time and that direction is not slowing down at all:

The 15th Amendment to the Constitution granted African American men the right to vote by declaring that the "right of citizens of the United States to vote shall not be denied or abridged by the United States or by any state on account of race, color, or previous condition of servitude." Although ratified on February 3, 1870, the promise of the 15th Amendment would not be fully realized for almost a century. Through the use of poll taxes, literacy tests and other means, Southern states were able to effectively disenfranchise African Americans. It would take the passage of the Voting Rights Act of 1965 before the majority of African Americans in the South were registered to vote.

The 1975 Amendments extended the provisions of the Voting Rights Act of 1965 for seven years. Established coverage for other minority groups including Native Americans, Hispanic Americans, and Asian Americans. Permanently banned literacy tests. Passed by the 94th Congress (1975–1977) as H.R. 6219.

Let's also see that on a Constitutional level there is precedence to loathe and consider it corrupt to restrict voters beyond them having the capacity to consent and be a Citizen of US (which they already are, I repeat that Prop is going to force US Citizens to do the US Naturalisation Test). Note that legislation that is made directly to assist the Constitution to be enacted in a just manner is still revolving around the Constitution and if it hasn't since been appealed is what the US currently still considers to be representing of Constitutional values:

In 1970, Congress voted to extend renewable portions of the Voting Rights Act for five more years. They also added some new provisions to the act.

The 1970 amendments included a nationwide ban on literacy tests and reduced residency requirements that could be applied in presidential elections.

The 1970 reauthorization also reduced the voting age [link to AGE subpage] in national elections from 21 to 18 years of age. Though this provision was overturned by the U.S. Supreme Court for non-federal elections, the voting age was permanently lowered by passage of the 26th Amendment in 1971.


Amendment 24 [source says XXIV but this is numerically same as 24]

ABOLITION OF POLL TAXES
Passed by Congress August 27, 1962. Ratified January 23, 1964

SECTION 1

The right of citizens of the United States to vote in any primary or other election for President or Vice President, for electors for President or Vice President, or for Senator or Representative in Congress, shall not be denied or abridged by the United States or any State by reason of failure to pay poll tax or other tax.

SECTION 2

The Congress shall have power to enforce this article by appropriate legislation.
The rule that, under the Equal Protection Clause of the Constitution, legislative voting districts must be the same in population size. The idea behind the rule is that one person’s voting power ought to be roughly equivalent to another person’s within the state. See Reynolds v. Sims, 377 U.S. 533 (1964).

Voter ID laws was basically the only thing Prop could bring forth in order to prove we restrict voters beyond having the capacity to consent and being a US Citizen. The reason their Prisoner Point was negated was not only that it's not clear at all which way the US is heading (towards allowing prisoners to vote or banning ex-cons from voting on top of Prisoners too or something in between) since it's State-level and everything that is State-level and open to variation across states is not Federal which means it's not at the Constitutional level of legislation or enforcement rendering the entire Voter ID and Prisoner issue both non-combatting of Constitutional values (which are held above all else in US law, I needn't prove that since Prop concedes it by never denying the importance of the Constitution).

What Voter ID laws were intended to do and legislated in order to allow was to prevent voter fraud meaning that anyone who can consent and was a US citizen had to attain and official ID card of some kind in order to be able to vote at the voting booth (back when the laws were made there was no Internet voting). This law ended up unfairly alienating voters of the races and/or ethnicities that I explicitly mentioned in my R3 quotes and expansion. This in no way at all disproves that voting in the US is for all who can consent and have proven to be valid US citizens (a prisoner can neither officially consent hence the Prison Rape Elimination Act and once released can be held to be less loyal and valid of a citizen than others until provably reformed over extensive periods of time). I don't have any idea at all how this disproves me on the basis of voting rights and accessibility to enact those rights for a consenting citizen in the US.

The entirety of Prop's case is one of taunting people of any outlandish view or who don't know something Prop considers too stupid to not know. Not only is this pure Ad Hominem by its very nature (they are too ignorant of this so therefore they are magically undeniably too stupid to vote accurately) but it's also lacking and entire third aspect to what makes a good candidate and what makes a good voter... Loyalty! I will just say this again:

Loyalty =/= Intelligence =/= Knowledge

That's the end of discussion really. Thanks for reading and have a nice life and enjoy your right to vote if you live in the US or somewhere where that right is well enforced and exercised! :)